LSAT and Law School Admissions Forum

Get expert LSAT preparation and law school admissions advice from PowerScore Test Preparation.

 Administrator
PowerScore Staff
  • PowerScore Staff
  • Posts: 8916
  • Joined: Feb 02, 2011
|
#23611
Complete Question Explanation

Strengthen—PR. The correct answer choice is (D)

This author discusses three different pesticides; for quick reference we can call them "T," "E," and "Z." The old belief was that T was more environmentally damaging than either E or Z. T was banned, and E and Z remained on the market. We now know that T is less harmful than either E or Z, so the author concludes that either T should be legalized, or E and Z should be outlawed.

The author objects to the inconsistency of legal pesticides which are more dangerous than certain illegal pesticides.

Answer choice (A): The author provides no information to assess the disparity between the two referenced legal pesticides, E and Z. All that we can say with certainty is that they are both more harmful than T, so this principle would not strengthen the author's assertion.

Answer choice (B): This answer choice is not applicable to the stimulus, because both of the referenced legal pesticides are harmful to the environment.

Answer choice (C): This principle does not strengthen the argument of the author, who is concerned with the inconsistency between harmful legal pesticides and a less harmful illegal pesticide.

Answer choice (D): This is the correct answer choice, and restates the prephrased principle above in more specific terms. Even more specifically, this choice refers to the fact that E and Z should be legal and T illegal only if E and Z are both less harmful to the environment than T. If this is valid, then the author's assertion is clearly bolstered.

Answer choice (E): Since the stimulus doesn't deal with any "harmless" pesticides, this cannot represent the principle
 Lawyered
  • Posts: 23
  • Joined: Jun 13, 2017
|
#36133
I was between answer choices D & E but I picked E because the stimulus does deal with "harmless" it says it was once thought the E & Z were "environmentally harmless" so...?? Or does the next part about greater harm of TSX mean E Z do less but still some harm? Ergo sum not harmless??
Administrator wrote:Complete Question Explanation

Strengthen—PR. The correct answer choice is (D)

This author discusses three different pesticides; for quick reference we can call them "T," "E," and "Z." The old belief was that T was more environmentally damaging than either E or Z. T was banned, and E and Z remained on the market. We now know that T is less harmful than either E or Z, so the author concludes that either T should be legalized, or E and Z should be outlawed.

The author objects to the inconsistency of legal pesticides which are more dangerous than certain illegal pesticides.

Answer choice (A): The author provides no information to assess the disparity between the two referenced legal pesticides, E and Z. All that we can say with certainty is that they are both more harmful than T, so this principle would not strengthen the author's assertion.

Answer choice (B): This answer choice is not applicable to the stimulus, because both of the referenced legal pesticides are harmful to the environment.

Answer choice (C): This principle does not strengthen the argument of the author, who is concerned with the inconsistency between harmful legal pesticides and a less harmful illegal pesticide.

Answer choice (D): This is the correct answer choice, and restates the prephrased principle above in more specific terms. Even more specifically, this choice refers to the fact that E and Z should be legal and T illegal only if E and Z are both less harmful to the environment than T. If this is valid, then the author's assertion is clearly bolstered.

Answer choice (E): Since the stimulus doesn't deal with any "harmless" pesticides, this cannot represent the principle
 Francis O'Rourke
PowerScore Staff
  • PowerScore Staff
  • Posts: 471
  • Joined: Mar 10, 2017
|
#36615
The stimulus states that although Envirochem and Zanar were once thought to be harmless, they do in fact cause harm to the environment; they cause greater harm than TSX-400 does. For that reason we can gather that either all three pesticides cause some amount of environmental harm or that Envirochem and Zanar definitely cause harm, and TSX-400 likely does.

Although the stimulus does use the phrase "environmentally harmless," the second half of the stimulus tells us that it is wrong to think of these chemicals as harmless. Since we do not have any chemicals that we know for sure are environmentally harmless, we cannot apply answer choice (E) to the argument.
 biskam
  • Posts: 124
  • Joined: Aug 18, 2017
|
#39381
How do you know to read that A is referring to (E and Z) instead of (T and Z) or (T and E)? I read the answer choice the latter two ways instead and thus it made it hard for me to choose between A and D because I felt as though they were almost saying the same thing.

Thanks!
 biskam
  • Posts: 124
  • Joined: Aug 18, 2017
|
#39382
follow up: I understand that in the stimulus the only two legal pesticides are E and Z but in my head part of the principle was that either E and Z should be banned or T be legalized, aka they can't all be legalized or all be banned.
Am I making sense/
 Eric Ockert
PowerScore Staff
  • PowerScore Staff
  • Posts: 164
  • Joined: Sep 28, 2011
|
#39761
Hi biskam!

The current situation the stimulus presents is:

E and Z are legal
T is illegal

The author is arguing for one of two changes:

1. E and Z are banned, resulting in:
T, E, & Z are all illegal

OR

2. T is legalized, resulting in:
T, E, & Z are all legal

The sufficient condition of answer choice (A) states that "if one (pesticide) is measurably more harmful to the environment than the other is..." Here, E is more harmful than T. Z is more harmful than T. So this rule would apply to either of those pairs. But the rule then goes on to say "...(those) two pesticides should not both be legal." Notice, however, that second option up above, from the conclusion, has all three being legal. That would not follow the rule in answer choice (A), so this answer won't work.

On the other hand, answer choice D is essentially saying that if you are going to make one pesticide legal while the other is illegal, then the legal one needs to be less harmful. The current (original) situation has the more harmful pesticides (E and Z) legal, while the less harmful pesticide is illegal. This violates the necessary condition of this rule. This would activate the contrapositive of the rule and tell you that the two pesticides should both be legal or both be illegal. So, to diagram:

One pesticide legal while other illegal :arrow: Legal pesticide less harmful

Contrapositive:

NOT Legal pesticide less harmful :arrow: NOT one pesticide legal while other illegal (both are legal or both are illegal)

This contrapositive would tell us that we must be left with all legal or all illegal pesticides. We can't have one of each with respect to T and E or T and Z.

Tricky question though. Hope that helps!
 cinnamonpeeler
  • Posts: 21
  • Joined: Apr 27, 2020
|
#75425
I know that on the LSAT, "either/or" should be read as "at least one of the two, possibly both." Is this meaning of the term present in this question, as well?

In this case, it is possible for both (1) and (2) to be adopted: we ban E and Z but we legalise TSX-400. This policy would suggest that we may only want to ban the most harmful pesticides, instead of all harmful or all extremely harmful pesticides.

Can someone also elaborate on why (C) is wrong?
 Adam Tyson
PowerScore Staff
  • PowerScore Staff
  • Posts: 5153
  • Joined: Apr 14, 2011
|
#76103
Correct, cinnamonpeeler - it is possible in this case that we might want to ban the new stuff and remove the ban on the old one, and the argument is that we have to do at least one of those two things. If this was a Could Be True question, an answer that said we should ban Envirochem and Zanar and remove the ban on TSX-400, that could be true and would be a good answer.

But this is a Strengthen question, and so we need to help the argument that we should either ban the new ones or else not ban the old one. Basically, what we want is see that if TSX-400 deserves to be banned, then so do the more harmful Envirochem and Zanar, and if Envirochem and Zanar do not deserve to be banned, then neither should TSX-400.

Answer C does not help because it is a Mistaken Reversal of something that would help. What we want is "if they are both harmful, they should both be illegal," but what answer C gives us is "if they are both illegal, they are both harmful." This is because the Necessary Condition indicator "only if" refers to them being harmful.

Get the most out of your LSAT Prep Plus subscription.

Analyze and track your performance with our Testing and Analytics Package.